[obm-l] Re: [obm-l] Quantos dígitos tem o fatorial de 7000?

2012-09-13 Por tôpico diego andres
Oi ennius,
A quantidade de digitos dependerá do número de fatores 2 e 5 que aparece na 
decomposição em fatores primos. Como num fatorial temos uma certa abundancia no 
número de fatores 2, o que determinará será o número de fatores 5.

1 - parte inteira de [7000/5] = 1400 (quantidade de numeros divisiveis por 5)
2 - parte inteira de [7000/25] =  280 (Contando o segundo fator dos numeros 
divisiveis por 25  --- * o primeiro ja foi contado em 1) 

3 - parte inteira de [7000/125] =  56 (Contando o terceiro fator dos numeros 
divisiveis por 125  --- * o primeiro ja foi contado em 1 e o segundo em 2) 
4 - parte inteira de [7000/625] =  11  
.
5 - parte inteira de [7000/3125] =  2 
...


S = 1400 + 280 + 56 + 11 + 2 = 1749

O caso geral voce deve fazer:

S = Somatorio(Parte inteira[ N / 5^i ] )   para i de 1 até infinito. 

O livro Teoria Elementar dos Numeros do Edmund Landau acho que ajudará você a 
entender melhor essa parte (Página 23 teorema 27 - e exemplo resolvido da 
pagina 25). Segue o link:
http://books.google.com.br/books?id=Q0wBV6wln3wCpg=PA11dq=teoria+elementar+dos+numeros+edmund+landausource=gbs_toc_rcad=4#v=onepageqf=false 


abs,
Diego Andrés



 De: ennius enn...@bol.com.br
Para: obm-l@mat.puc-rio.br obm-l@mat.puc-rio.br 
Enviadas: Quinta-feira, 13 de Setembro de 2012 10:27
Assunto: [obm-l] Quantos dígitos tem o fatorial de 7000?
 
Prezados Colegas,

Qual o melhor método para calcular quantos dígitos tem o fatorial de 7000 (ou 
de qualquer outro número natural grande)?

Desde já, muito obrigado.

Ennius Lima
=
Instruções para entrar na lista, sair da lista e usar a lista em
http://www.mat.puc-rio.br/~obmlistas/obm-l.html
=

Re: [obm-l] Apostol - Continuidade

2007-05-27 Por tôpico diego andres
letra a :
como:   0 = | f(u)-f(v) | = |u-v|
se u-v implica0= |f(u)-f(v)|= 0  -   f(u)=f(v)   
(confronto).
letra b:
TEM UMA PARTE ERRADA :
b)Considerando f integrável em [a,b], prove que
|(integral de a ate b f(x)dx) - (b-a)f(x)| = ((b-a)^2)/2

é na verdade:

b)Considerando f integrável em [a,b], prove que 
| (integral de a ate b f(x)dx) - (b-a)f(a) | = ((b-a)^2)/2  

faz uv sem  perda de generalidade , e faz  u - v = x  logo:
| f(v+x)-f(v) | = x   então   
dividimos em dois casos..
1- f(v+x)-f(v)  = x
fazendo v=a
f(x+a)-f(a)=x
integrando de b-a até zero
 integral de b-a até  0 ( f(x+a) )  -  (b-a)f(a) = ( (b-a)^2)  /2
translatando  a integral  por a.  
(integral de a ate b f(x)dx) - (b-a)f(a) = ((b-a)^2)/2
2-   f(v)- f(v+x)  = x
no outro caso chegaremos a :
 (b-a)f(x) - (integral de a ate b f(x)dx)  =  ((b-a)^2)/2

assim concluimos a nossa demonstração.
 tem ainda uma letra c que voçê não botou mais a ideia é a mesma da letra b

Adriano Torres [EMAIL PROTECTED] escreveu: Seja f uma função tal que |f(u) - 
f(v)| = |u-v|, para todo u e v no 
intervalo [a,b].
a)Prove que f é continua em cada ponto de [a,b]
b)Considerando f integrável em [a,b], prove que

|(integral de a ate b f(x)dx) - (b-a)f(x)| = ((b-a)^2)/2

Valeu pela ajuda. Esse livro é tenso!

_
O Windows Live Spaces é seu espaço na internet com fotos (500 por mês), blog 
e agora com rede social http://spaces.live.com/

=
Instruções para entrar na lista, sair da lista e usar a lista em
http://www.mat.puc-rio.br/~nicolau/olimp/obm-l.html
=


 __
Fale com seus amigos  de graça com o novo Yahoo! Messenger 
http://br.messenger.yahoo.com/ 

Re: [obm-l] Congruência

2007-03-04 Por tôpico diego andres
777 eh congruo a 7 mod 10
  logo (777)^2 eh congruo a 49 que eh congruo a -1 mod 10
  então  [(777)^2]^388 = 777^776 eh congruo a 1 mod 10
= 777^777 eh congruo a 7 mod 10.
Josh Rodrigues [EMAIL PROTECTED] escreveu:
  Olá, estive dando uma olhada no site do grupoteorema que citaram 
anteriormente aqui na lista e vi um artigo sobre congruências. Fiquei 
interessado pois ouvi dizer que essa ferramenta ajuda bastante a resolver e 
provar vários problemas. Só que eu não entendi como se aplica, li as 
propriedades mas não entendi muito bem como usá-las. No primeiro problema 
diz Qual o último dígito de 777^777?. Gostaria de ter esse problema 
resolvido para que eu entenda como se aplica essas propriedades.

Obrigado pela atenção.

_
Verificador de Segurança do Windows Live OneCare: combata já vírus e outras 
ameaças! http://onecare.live.com/site/pt-br/default.htm

=
Instruções para entrar na lista, sair da lista e usar a lista em
http://www.mat.puc-rio.br/~nicolau/olimp/obm-l.html
=


 __
Fale com seus amigos  de graça com o novo Yahoo! Messenger 
http://br.messenger.yahoo.com/ 

Re: [obm-l] homomorfismo e isomorfismo!!!

2006-12-31 Por tôpico diego andres
valeu LEX(soh 2horas pra eu intender)!!!
feliz ano novo pra tu

Alex pereira Bezerra [EMAIL PROTECTED] escreveu: Bem vc pode pensar da 
seguinte forma.Sejam A e B anéis.Um homomorfismo
de A em B é uma aplicação fi de A em B tal que fi(1) = 1 e para todo
x,y e z pertencentes a A temos fi(x + yz)= fi(x) + fi(y).fi(z).
Dizemos que um homomorfismo bijetivo fi é um isomorfismo;neste caso,a
aplicação inversa é necessariamente um homomorfismo.Dizemos que os
anéis A e B são isomorfos se existir um isomorfismo fi: A em B.Nota:
Um homomorfismo sobrejetor é chamado epimorfismo.Espero ter
ajudado.Caso vc queira exemplos é só
pedir__

=
Instruções para entrar na lista, sair da lista e usar a lista em
http://www.mat.puc-rio.br/~nicolau/olimp/obm-l.html
=


 __
Fale com seus amigos  de graça com o novo Yahoo! Messenger 
http://br.messenger.yahoo.com/ 

[obm-l] homomorfismo e isomorfismo!!!

2006-12-29 Por tôpico diego andres
Alguem se sujeitaria a explicar de forma mais simpels o possivel a definicao de 
um mapa com homomorfismo.E tambem se ele tiver isomorfismo.
   
  Desde já grato.

 __
Fale com seus amigos  de graça com o novo Yahoo! Messenger 
http://br.messenger.yahoo.com/ 

Re: [obm-l] OFFTOPIC - Quimica - Quem puder ajudar, agradeco

2006-08-12 Por tôpico diego andres
você poderia aumentar a temperatura da soluçao ja que a curva de solubilidade do enxofre é endotermica,ou seja ,maior temperatura vai favorecer uma maior solubilidade[""]Diego Andrés"Salhab [ k4ss ]" [EMAIL PROTECTED] escreveu: Olá,sei que nao é o objetivo desta lista, mas estou precisando de uma ajuda com quimica, e, como sei que muitos aqui nao sao necessariamente matematicos, envio esta mensagem.o enxofre é insoluvel em agua, porem, preciso solubiliza-lo de algum modo.alguem sabe como fazer isso?existe algum outro composto que eu posso colocar na agua para que essa solubilizacao seja possivel?agradeco qualquer ideia..abracos,Salhab=Instruções para entrar na lista, sair da lista e usar a
 lista emhttp://www.mat.puc-rio.br/~nicolau/olimp/obm-l.html= 
		 
Novidade no Yahoo! Mail: receba alertas de novas mensagens no seu celular. Registre seu aparelho agora!

[obm-l] soma das n-esimas potencias....

2006-07-27 Por tôpico diego andres
Eu sei que :1+2+3+...+n=n(n+1)/21²+2²+3²+...+n²=n(n+1)(2n+1)/6.o método que eu sei eh muita conta ,para saber a soma das k-ésimas potências eu teria que saber a da j-ésimas potências de j=k-1 até j=1ai quando k é muito grande não vale apena esse método...me disseram que existe a soma das n-ésima potências em função de n(parece que eh soma de bernoulli)...alguem pode me mostrar esse jeito por favor ![""]Diego Andrés 
		 
Novidade no Yahoo! Mail: receba alertas de novas mensagens no seu celular. Registre seu aparelho agora!

Re: [obm-l] Desigualdade com Pi

2006-07-25 Por tôpico diego andres
uma outra forma de chegar em pi é que :zeta(2)=pi²/6 *obs:( zeta(2)=1/1²+1/2²+1/3²+1/4²+...+1/n²+. )então se somar alguns termos chega com uma boa aproximacao que pi=3.1415(agora se somarmos no computador varios termos chegamos a uma boa quantidade de casas)Rogerio Ponce [EMAIL PROTECTED] escreveu: Ola' Claudio e Bernardo,nao gostei da solucao 'na marra' porque o valor de PIfoi tirado da cartola. Como provar que PI vale3.141592653...?Por outro lado, nao precisava de tanto pra mostrar que1.414 * 1.414  21.732 * 1.732  3De onde sqrt(2) + sqrt(3)  3.146 , que 'deve sermaior que PI' - foi isto que tentei provar quandotambem resolvi 'fazer na marra'...Eu ja' havia tentado sair por 'n*tan(pi/n)' , usando
 o'arco-metade' sucessivamente, a partir de pi/4 ou depi/6 . Mas, como isso passa a valer somente para n47,a expressao final e' cavalar. E ainda por cima os doistermos principais do numerador sempre sao umadiferenca, embora eu quisesse obter uma soma 'com carade sqrt(2) + sqrt(3)' para ajudar na simplificacao.Tambem tentei usar alguma integral que o resultadofosse uma fracao de pi, ou de tg(pi/n) . Entao,alterando 'conveniente' o integrando, talvez fossepossivel obter sqrt(2)+sqrt(3) , ou alguma coisaintermediaria, para o mesmo intervalo. Mas tambem naoconsegui.Entao apelei para a soma dos termos da serie(-1)^(n-1) * n^(-6) , que, para n de 1 em diante, vale(31/30240)*pi^6Assim, com os 3 primeiros termos, podemos dizer:1/1 - 1/64 + 1/729  (31/30240) * pi^6de onde, pi  3.142 .Ficou muito feia, mas ate' agora nao consegui nadamelhor...[]s,Rogerio
 Ponce--- "claudio.buffara" <[EMAIL PROTECTED]>escreveu: Bem, eu estava me referindo a uma demonstracao geometrica ou trigonometrica com um minimo de elegancia (com todo o respeito a sua solucao, claro!) ...  A aproximacao Pi ~ raiz(2) + raiz(3) eh bastante boa. A diferenca eh de apenas 0.00467..., ou seja, menos de 0,15%. Ao aproximar Pi por excesso por meio do semi-perimetro (ou da area) de um poligono regular (e convexo) circunscrito ao circulo unitario, esta precisao soh eh ultrapassada quando o numero de lados eh = 48. Ou seja, 47*tan(Pi/47)  raiz(2) + raiz(3)  48*tan(Pi/48)  Pi. Isso talvez signifique que uma demonstracao puramente geometrica nao eh muito trivial.  []s, Claudio.  De:[EMAIL PROTECTED] 
 Para:obm-l@mat.puc-rio.br  Cópia:  Data:Sat, 15 Jul 2006 17:33:12 +0200  Assunto:Re: [obm-l] Casa de Pombos e Desigualdade com Pi   Viva as férias (até que enfim)   Bom, o seu PCP ainda nao foi, mas pra \pi (estilo "NA MARRA"):  Eleve ao quadrado (todo mundo é positivo):  2 + 2 raiz(6) + 3  Pi^2 = 2 raiz(6) = Pi^2 - 5   E mais uma vez (notar que Pi  3 = Pi^2  9  5):  24  Pi^4 - 10Pi^2 + 25 =  0  Pi^4 - 10 Pi^2 + 1   Agora calcule as raízes de x^4 - 10x^2 + 1 ...  x^2 = 5 +/- raiz(25 + 1) = apenas duas raizes, as da raiz positiva do quadrado  x^2 = 10 + um pouquinho   Agora saiba que Pi = 3.14159265358979... e que raiz(10) = 3.16.. e pronto:  as
 raizes do polinômio sao maiores do que +- Pi, e portanto o valor em  Pi é menor do que zero pois o coeficiente de segundo grau é positivo.   Uma calculadora dá:  sqrt(2) + sqrt(3) - %pi  ans = 0.0046717   T+,  --  Bernardo Freitas Paulo da CostaOn 7/15/06, claudio.buffara wrote: Esse tah me enchendo o saco: Prove que toda sequencia de 2n-1 inteiros (nao necessariamente distintos)   possui uma subsequencia de n inteiros cuja soma eh divisivel por n. *** Ha alguns meses alguem mandou pra lista o problema de se provar que:   raiz(2) + raiz(3)  Pi.   Foi enviada alguma solucao? 
[]s, Claudio.  ___ Novidade no Yahoo! Mail: receba alertas de novas mensagens no seu celular. Registre seu aparelho agora! http://br.mobile.yahoo.com/mailalertas/  =Instruções para entrar na lista, sair da lista e usar a lista emhttp://www.mat.puc-rio.br/~nicolau/olimp/obm-l.html= 
		 
Novidade no Yahoo! Mail: receba alertas de novas mensagens no seu celular. Registre seu aparelho agora!

[obm-l] numeros perfeitos

2006-07-25 Por tôpico diego andres
gostaria de saber como provar que todo numero perfeito nunca pode ser quadrado perfeito 
		 
O Yahoo! está de cara nova. Venha conferir!

Re: [obm-l] Desigualdade com Pi

2006-07-25 Por tôpico diego andres
é realmente, pelo fato de a ter uma potencia sexta ela converge primeiro por isso é bem melhor...Rogerio Ponce [EMAIL PROTECTED] escreveu: Ola' Diego, ja' que a inequacao e' sabidamente verdadeira, o interessante deste problema e' arranjar uma demonstracao que possa ser feita 'na mao' . Isto significa que as contas nao devem ultrapassar as 4 operacoes, e que o processo nao deve ser longo.  Entretanto, se voce quiser usar a soma dos termos da serie 1/n^2 , mais de mil termos sao necessarios para se chegar `a casa dos milesimos correta. Fora o fato de que voce estara' chegando a PI com uma aproximacao "por baixo" . E no nosso problema, voce precisa chegar com uma aproximacao "por cima".  Repare que, na serie que eu sugeri, voce consegue isso com apenas 3 termos. Mas outras series parecidas (de ordem
 superior) tambem poderiam ser usadas. Mas, como em todas elas, os 3 primeiros termos eram necessarios, preferi usar a de menor ordem.  Grande abraco, Rogerio Ponce  PS: ontem, um colega me perguntou "afinal, qual a dificuldade em se provar que raiz(2) + raiz(3)   PI , ja' que  raiz(2)  1.414 , raiz(3)  1.732 , e 3.146  3.14159...= PI " ?  Obviamente mostrar que 3.146  3.14159... nao tem nada de mais. A questao aqui e' como calcular PI ( em vez de "olhar o valor de PI" ), com uma precisao suficiente que nos permita fazer a afirmacao original.   diego andres [EMAIL PROTECTED] escreveu: uma outra forma de chegar em pi é que :zeta(2)=pi²/6 *obs:(
 zeta(2)=1/1²+1/2²+1/3²+1/4²+...+1/n²+. )então se somar alguns termos chega com uma boa aproximacao que pi=3.1415(agora se somarmos no computador varios termos chegamos a uma boa quantidade de casas)Rogerio Ponce [EMAIL PROTECTED] escreveu: Ola' Claudio e Bernardo,nao gostei da solucao 'na marra' porque o valor de PIfoi tirado da cartola. Como provar que PI vale3.141592653...?Por outro lado, nao precisava de tanto pra mostrar que1.414 * 1.414  21.732 * 1.732  3De onde sqrt(2) + sqrt(3)  3.146 , que 'deve sermaior que PI' - foi isto que tentei provar quandotambem resolvi 'fazer na marra'...Eu ja' havia tentado sair por 'n*tan(pi/n)' , usando  o'arco-metade' sucessivamente, a partir de pi/4 ou depi/6 . Mas, como isso passa a valer somente
 para n47,a expressao final e' cavalar. E ainda por cima os doistermos principais do numerador sempre sao umadiferenca, embora eu quisesse obter uma soma 'com carade sqrt(2) + sqrt(3)' para ajudar na simplificacao.Tambem tentei usar alguma integral que o resultadofosse uma fracao de pi,  ou de tg(pi/n) . Entao,alterando 'conveniente' o integrando, talvez fossepossivel obter sqrt(2)+sqrt(3) , ou alguma coisaintermediaria, para o mesmo intervalo. Mas tambem naoconsegui.Entao apelei para a soma dos termos da serie(-1)^(n-1) * n^(-6) , que, para n de 1 em diante, vale(31/30240)*pi^6Assim, com os 3 primeiros termos, podemos dizer:1/1 - 1/64 + 1/729  (31/30240) * pi^6de onde, pi  3.142 .Ficou muito feia, mas ate' agora nao consegui nadamelhor...[]s,Rogerio  Ponce--- "claudio.buffara" escreveu: Bem, eu estava me referindo a uma demonstracao geometrica ou trigonometrica com um minimo de elegancia (com todo o respeito a sua solucao, claro!) ...  A aproximacao Pi ~ raiz(2) + raiz(3) eh bastante boa. A diferenca eh de apenas 0.00467..., ou seja, menos  de 0,15%. Ao aproximar Pi por excesso por meio do semi-perimetro (ou da area) de um poligono regular (e convexo) circunscrito ao circulo unitario, esta precisao soh eh ultrapassada quando o numero de lados eh = 48. Ou seja, 47*tan(Pi/47)  raiz(2) + raiz(3)  48*tan(Pi/48)  Pi. Isso talvez signifique que uma demonstracao puramente geometrica nao eh muito trivial.  []s, Claudio.  De:[EMAIL PROTECTED]   Para:obm-l@mat.puc-rio.br  Cópia: 
 Data:Sat, 15 Jul 2006 17:33:12 +0200  Assunto:Re: [obm-l] Casa de Pombos e Desigualdade com Pi   Viva as férias (até que enfim)   Bom, o seu PCP ainda nao foi, mas pra \pi (estilo "NA MARRA"):  Eleve ao quadrado (todo mundo é positivo):  2 + 2 raiz(6) + 3   Pi^2 = 2 raiz(6) = Pi^2 - 5   E mais uma vez (notar que Pi  3 = Pi^2  9  5):  24  Pi^4 - 10Pi^2 + 25 =  0  Pi^4 - 10 Pi^2 + 1   Agora calcule as raízes de x^4 - 10x^2 + 1 ...  x^2 = 5 +/- raiz(25 + 1) = apenas duas raizes, as da raiz positiva do quadrado  x^2 = 10 + um pouquinho   Agora saiba que Pi = 3.14159265358979... e que raiz(10) = 3.16.. e pronto:  as  raizes do polinômio sao maiores do que +- Pi, e portanto o
 valor em  Pi é menor do que zero pois o coeficiente de segundo grau é positivo.   Uma calculadora dá:  sqrt(2) + sqrt(3) - %pi  ans = 0.0046717   T+,  --  Bernardo Freitas Paulo da CostaOn 7/15/06,  claudio.buffara wrote: Esse tah me enchendo o saco: Prove que toda sequencia de 2n-1 inteiros (nao necessariamente distintos)   possui uma subsequencia de n inteiros cuja soma eh divisivel por n. *** Ha alguns meses alguem mandou pra lista o problema de se provar que:   raiz(2) + raiz(

Re: [obm-l] Desigualdade

2006-07-24 Por tôpico diego andres
Bem, eu acho que eh assim:z²=1/(xy)²usa M.A.=M.G. em: x²+4y²+2/(xy)²+4xy =4(32xy)^(1/4)=8(2xy)^(1/4)como essa ultima parte tem que ser inteira, já que partimos de soma de inteiros,xy=8e o maior valor é 16.vê se bate com a resposta por favor[""]Diego AndrésKlaus Ferraz [EMAIL PROTECTED] escreveu: Sejam x, y, e z inteiros positivos tal que xyz=32 calcule o menor valor da expressao x^2+4y^2+2z^2+4xy. Alguem tem alguma ideia esperta?  Yahoo! Search  Música para ver e ouvir: You're Beautiful, do James
 Blunt 
		 
Novidade no Yahoo! Mail: receba alertas de novas mensagens no seu celular. Registre seu aparelho agora!

[obm-l] Desigualdades do rearranjo!!!

2006-07-21 Por tôpico diego andres
Demonstre que:1)x²+y²+x² = xy+yz+zx2)x³+y³+x³ = x²y+y²z+z²x.eu nao consigo mostrar porque o rearranjo ,nesse caso, fuciona grato:Diego Andrés  
		 
Você quer respostas para suas perguntas? Ou você sabe muito e quer compartilhar seu conhecimento? Experimente o Yahoo! Respostas!

[obm-l] desigualdades!!!

2006-06-29 Por tôpico diego andres
gostaria que alguem tambem resolvesse mais essas questoes: 1) para quaisquer reais positivos a,b,c mostre que : c(a²-ab+b²)^(1/2)+a(b²-bc+c²)^(1/2) =b(a²+ac+c²)^(1/2)2)se somatorio de xi²=1(sao reais positivos) entao determine o valor maximo de:   nn somatorio[ (xi^5)/( (somatorio xj) - xi ) ]i=1j=1 
		 
Abra sua conta no Yahoo! Mail - 1GB de espaço, alertas de e-mail no celular e anti-spam realmente eficaz. 

[obm-l] desigualdades!!!

2006-06-29 Por tôpico diego andres
gostaria que alguem tambem resolvesse mais essas questoes: 1) para quaisquer reais positivos a,b,c mostre que : c(a²-ab+b²)^(1/2)+a(b²-bc+c²)^(1/2) =b(a²+ac+c²)^(1/2)2)se somatorio de xi²=1(sao reais positivos) entao determine o valor maximo de:   nn somatorio[ (xi^5)/( (somatorio xj) - xi ) ]i=1j=1 
		 
Novidade no Yahoo! Mail: receba alertas de novas mensagens no seu celular. Registre seu aparelho agora!

[obm-l] desigualdades....

2006-06-22 Por tôpico diego andres
alguem se habilita a resolver esses problemas pra mim:1) prove que para todos a,b,c reais positivos temos: 1/(a³+b³+abc) +1/(b³+c³+abc) + 1/(c³+a³+abc)=1/(abc)2)sejam a,b,c reais dados.Prove que : a³/(a²+ab+b²)+b³/(b²+bc+c²)+c³/(c³+ac+a²) = (a+b+c)/3 
		 
Yahoo! Search 
Música para ver e ouvir: You're Beautiful, do James Blunt

[obm-l] OBI !!!

2006-05-15 Por tôpico diego andres
Apesar de nao ser o foco da lista,gostaria de saber se alguem fez a segunda fase da olimpiada brasileira de computacao.Se alguem fez comente comigo o que voce achou dela.grato: Diego
		 
Abra sua conta no Yahoo! Mail - 1GB de espaço, alertas de e-mail no celular e anti-spam realmente eficaz. 

[obm-l] eureka 23 !!!

2006-04-19 Por tôpico diego andres
Alguem sabe quando sairá a eureka 23 ??
		 
Abra sua conta no Yahoo! Mail - 1GB de espaço, alertas de e-mail no celular e anti-spam realmente eficaz. 

Re: [obm-l] combinação linear (acho)ajuda

2006-04-05 Por tôpico diego andres
ei wellington: 2x+3y+4z=13,5 (vezes k) 3x+2y+z=8,5  (vezes j) 1x+1y+1z=? se x+y+z for a combinacao linear das duas outras equaçoes,teremos : 2k+3j=1 3k+2j=14k+j=1 resolvendo esse sistema: k=j=1/5 x+y+z=13,5/5+8,5/5=2,7+1,7=4,4(letra b)  [EMAIL PROTECTED] escreveu:  Pessoal gostaria de ajuda,2colheres +3garfos+4facas = R$13,50 3colheres+2garfos+1faca = R$8,50
  1colhere + 1garfo +faca = ? me ajudem pessoal,eu fiz por teste e deu R$3,60 a)3,60 b)4,40 c) 5,30 d) 6,20 e)7,00  Atenciosamente wellingtonTry the New Netscape Mail Today! Virtually Spam-Free | More Storage | Import Your Contact Listhttp://mail.netscape.com  
		 
Yahoo! doce lar. Faça do Yahoo! sua homepage.

Re: [obm-l] errata:numeros primos=fisica quantica!!!

2006-03-29 Por tôpico diego andres
O problema desse artigo eh que não entra em detalhes na conexão. mas,eu acho que o autor desse artigo não foi relmente o autor do trabalho e sim um jornalista curioso!!!Por isso que não está bem explicado.É uma espécie de "fofoca" da descoberta.Ronaldo Luiz Alonso [EMAIL PROTECTED] escreveu:   Sim, mas o artigo não fala nada sobre como essa  conexão  é feita . Onde entra a função zeta de  Riemann? Eles dizem que conseguem uma fórmula para prever todos   os primos em sequência. A pergunta é como 
 ?...- Original Message -    From:diego andresTo: obm-l@mat.puc-rio.brSent: Tuesday, March 28, 2006 10:36PM   Subject: [obm-l] errata:numerosprimos=fisica quantica!!!   me desculpem pessoal eu mandei o link errado mais o certoé:http://www.seedmagazine.com/news/2006/03/prime_numbers_get_hitched.php?utm_source=seedmag-main=rsspage=3 !
Yahoo!
 SearchDê uma espiadinha e saiba tudo sobre o BigBrother Brasil.
		 
Abra sua conta no Yahoo! Mail - 1GB de espaço, alertas de e-mail no celular e anti-spam realmente eficaz. 

RE: [obm-l] PA e primos

2006-03-29 Por tôpico diego andres
eu acho que quando se considerar r=1modp voçe particulariza e se r for impar(p diferente de 2).Tem que ser posto um caso mais geral no qual para qualquer r o termo procurado será composto.E a primeira aparição será a_(a1+1) mesmo.Qwert Smith [EMAIL PROTECTED] escreveu: O limite superior e provavelmente o da sua prova.a_(a1+1) = a1 + a1 * r = a1 * ( r + 1 ) e nao primoMas esse e o pior caso possivel.  Seja p um primo tal que r = 1 mod pEntao o primeiro termo composto sera a1 + n * r onde n = p - [a1 mod p]foi o que me veio a cabeca ate agoraFrom: Iuri Reply-To: obm-l@mat.puc-rio.brTo: obm-l@mat.puc-rio.brSubject: [obm-l] PA e primosDate: Wed, 29 Mar 2006 10:24:50 -0300Vi uma questão esses dias, e !
fiquei
 curioso.Em uma PA infinita, com termo inicial a1 natural e razão r natural ediferente de zero. Prove que esta PA não pode ser composta apenas pornumeros primos.Consegui provar, mas fiquei um tempão pensando: em qual termo exatamenteacontece a primeira ocorrencia de um numero nao primo, em função de r e a1?Isso é uma questao razoavel ou vai ter q desenvolver mta coisa pra chegar noresultado?Iuri=Instruções para entrar na lista, sair da lista e usar a lista emhttp://www.mat.puc-rio.br/~nicolau/olimp/obm-l.html=
		 
Yahoo! Search 
Imposto de Renda 2006: o prazo está acabando. Faça já a sua declaração no site da Receita Federal.

[obm-l] primos=física quantica

2006-03-28 Por tôpico diego andres
leiam esse artigo sobre numeros primos terem ligaçoes com fisica quantica:  http://br.f361.mail.yahoo.com/ym/Compose?YY=31733order=downsort=datepos=0view=ahead=b 
		 
Yahoo! Search 
Dê uma espiadinha e saiba tudo sobre o Big Brother Brasil.

[obm-l] errata:numeros primos=fisica quantica!!!

2006-03-28 Por tôpico diego andres
me desculpem pessoal eu mandei o link errado mais o certo é:  http://www.seedmagazine.com/news/2006/03/prime_numbers_get_hitched.php?utm_source=seedmag-main=rsspage=3 
		 
Yahoo! Search 
Dê uma espiadinha e saiba tudo sobre o Big Brother Brasil.

[obm-l] En: funçao geradora ordinaria!!!

2006-02-05 Por tôpico diego andres
Observação: mensagem anexa encaminhada.
		 
Yahoo! Acesso Grátis - Internet rápida e grátis. Instale o discador agora!---BeginMessage---
gostaria que alguem achasse a funcao geradora da sequencia(1,1,1,0,0,0,1,1,1,0,0,0...). grato Diego Andrés 
		 
Yahoo! doce lar. Faça do Yahoo! sua homepage.---End Message---


Re: [obm-l] Re: [obm-l] Re: [obm-l] Re: [obm -l] funçao geradora ordinaria!!!

2006-02-02 Por tôpico diego andres
caros marcelo e henrique,essa funçao não pode ser polinomial ,pois para um "x" diferente tera mais de um "y" igual(e pela definiçao isso não é uma funçao).Eu queria que voces achassem a funçao ordinaria.EX: "infinito" produtorio (1+x^(i^2)) i=0 vai gerar todos os numeros que podem ser escritos como a soma de quadrados distintos.  Marcelo Salhab Brogliato [EMAIL PROTECTED] escreveu: Olá,menor ou igual.. rs :)realmente, acabei de ver que digitei faltando o igual!É.. sua função também ficou bem mais simples.Bem legal!Abraços,Salhab- Original Message - From: "Henrique Rennó" To: Sent: Wednesday, February 01, 2006 1:24 PMSubje!
ct: Re:
 [obm-l] Re: [obm-l] Re: [obm-l] funçao geradora ordinaria!!!Olá Marcelo!!!A função [x] que você definiu é "maior inteiro menor que x" ou "maiorinteiro menor ou igual a x"???Acredito que a fórmula que passei também funciona, pois procurei gerarnúmeros naturais ímpares e pares de três em três:1,1,1,2,2,2,3,3,3,4,4,4,... . Dessa forma, ao efetuar mod 2 temosapenas 1 ou 0, gerando assim a seqüência.AbraçosOn 2/1/06, Marcelo Salhab Brogliato  wrote: Olá, pode ser que esteja, suas contas para f(0) estavam quase totalmente corretas. Vc obteve: 2 * 1/2 = 1 .. e não 0. Plotei o grafico usando o Graphmatica e obtive a sequencia pedida.. posso ter errado algo. Abraços, Salhab - Original Message - From: "Henrique Rennó"  To:  Sent: T!
uesday,
 January 31, 2006 4:31 PM Subject: Re: [obm-l] Re: [obm-l] funçao geradora ordinaria!!! Olá Marcelo!!! Acredito que a fórmula encontrada não está correta. Caso eu esteja errado me corrija. f(x) = [ 2 ( (x+3/2)/3 - [ (x+3/2)/3 ] ) ] f(0): 0+3/2 = 3/2 3/2/3 = 1/2 [x] é o maior inteiro menor que x [1/2] = 0 f(0) = [2(1/2 - 0)] = 0 -- valor incorreto f(1): 1+3/2 = 5/2 5/2/3 = 5/6 [x] é o maior inteiro menor que x [5/6] = 0 f(0) = [2(5/6 - 0)] = 1 -- valor correto f(2): 2+3/2 = 7/2 7/2/3 = 7/6 [x] é o maior inteiro menor que x [7/6] = 1 f(0) = [2(7/6 - 1)] = 0 -- valor incorreto Acho que a fórmula abaixo pode ser usada: f(x) = [(x+3)/3] mod 2, onde x pertence a N e [x] é o maior inteiro menor !
ou igual
 a x f(0) = [(0+3)/3] mod 2 = [3/3] mod 2 = 1 mod 2 = 1 f(1) = [(1+3)/3] mod 2 = [4/3] mod 2 = 1 mod 2 = 1 f(2) = [(2+3)/3] mod 2 = [5/3] mod 2 = 1 mod 2 = 1 f(3) = [(3+3)/3] mod 2 = [6/3] mod 2 = 2 mod 2 = 0 f(4) = [(4+3)/3] mod 2 = [7/3] mod 2 = 2 mod 2 = 0 f(5) = [(5+3)/3] mod 2 = [8/3] mod 2 = 2 mod 2 = 0 f(6) = [(6+3)/3] mod 2 = [9/3] mod 2 = 3 mod 2 = 1 f(7) = [(7+3)/3] mod 2 = [10/3] mod 2 = 3 mod 2 = 1 f(8) = [(8+3)/3] mod 2 = [11/3] mod 2 = 3 mod 2 = 1 f(9) = [(9+3)/3] mod 2 = [12/3] mod 2 = 4 mod 2 = 0 f(10) = [(10+3)/3] mod 2 = [13/3] mod 2 = 4 mod 2 = 0 f(11) = [(11+3)/3] mod 2 = [14/3] mod 2 = 4 mod 2 = 0 f(12) = [(12+3)/3] mod 2 = [15/3] mod 2 = 5 mod 2 = 1 . . . Abraços, On 1/31/06, Marcelo Salhab Brogliato  wrote:  Olá,  então, fiz o seguinte:!
; 
 f(x) = x - [x] , onde [x] é o maior inteiro menor que x   Fiz o seguinte, fiz o grafico ir até 2, ao invés de 1..  f(x) = 2(x - [x])   Então, estiquei para que ao inves do periodo ser 1, ser 3.  Então:   f(x) = 2(x/3 - [x/3])   Assim, g(x) = [f(x)] = [2(x/3 - [x/3])], x pertence aos naturais  nos da a seguencia:  g(0) = 0  g(1) = 0  g(2) = 0  g(3) = 1  g(4) = 1  g(5) = 1  g(6) = 0  e assim segue..  agora transladamos o grafico para tras..  logo:   f(x) = [ 2 ( (x+3/2)/3 - [ (x+3/2)/3 ] ) ]  nos da a sequencia desejada.   Abraços,  Salhab   - Original Message -  From: diego andres  To:
 obm-l@mat.puc-rio.br  Sent: Tuesday, January 31, 2006 10:50 AM  Subject: [obm-l] funçao geradora ordinaria!!!gostaria que alguem achasse a funcao geradora da  sequencia(1,1,1,0,0,0,1,1,1,0,0,0...).  grato Diego Andrés  Yahoo! doce lar. Faça do Yahoo! sua homepage.   -- Henrique = Instruções para entrar na lista, sair da lista e usar a lista em http://www.mat.puc-rio.br/~nicolau/olimp/obm-l.html = = Instruções para entrar na lista, sair da lista e usar a lista em
 http://www.mat.puc-rio.br/~nicolau/olimp/obm-l.html =--Henrique=Instruções para entrar na lista, sair da lista e usar a lista emhttp://www.mat.puc-rio.br/~nicolau/olimp/obm-l.html= =Instruções para entrar na lista, sair da lista e usar a lista emhttp://www.mat.puc-rio.br/~nicolau/olimp/obm-l.html=
		 
Yahoo! doce lar. Faça do Yahoo! sua homepage.

[obm-l] funçao geradora ordinaria!!!

2006-01-31 Por tôpico diego andres
gostaria que alguem achasse a funcao geradora da sequencia(1,1,1,0,0,0,1,1,1,0,0,0...). grato Diego Andrés 
		 
Yahoo! doce lar. Faça do Yahoo! sua homepage.

Re: [obm-l] Livros de Física

2006-01-23 Por tôpico diego andres
o topicos da fisica é ideal para o ensino médiosaulo nilson [EMAIL PROTECTED] escreveu: o bonjorno era muito legal,fundamentos da fisica, alicerces, Paraná. On 1/22/06, Pierry Ângelo Pereira [EMAIL PROTECTED] wrote:  Fundamentos de matemática elementar está para matemática no ensino médio, assim como _ está para física no ensino médio.  Alguém me recomendar uma boa coleção que se encaixe ali?  Outra coisa, quem tiver os videos do http://strato.impa.br(cursos de aperfeiçoamento de professores no impa), e puder me vender em dvd ou cd's, a situação é que não tenho internet banda larga e preciso bastante daqueles vídeos.   Por favor quem tiver me informe.  Abraços,  Pierry Ângelo Pereira 
		 
Yahoo! doce lar. Faça do Yahoo! sua homepage.

[obm-l] funçao geradora!!!

2006-01-23 Por tôpico diego andres
achar a funçao geradora de (1,1,1,0,0,0...) . agradecidamente Diego Andrés 
		 
Yahoo! doce lar. Faça do Yahoo! sua homepage.

[obm-l] duvida(raizes primitivas)!!!

2006-01-14 Por tôpico diego andres
gostaria de saber se raizes primitivas incongruentes sao aquela que possuem a mesma base ex:  2eh raiz primitiva modulo5 logo 2^3 tb eh  7eh raiz primitiva modulo 5 logo 7^3 tb ehpq tem um teorema que diz que existe exatamentefi(fi(n))raizes primitivas incongruentes onde fi eh a funçao de euler logo fi(fi(5))=2 e ja existe 4 raizes primitivas como mostrado no exemplo.Eu pensei que para cada raiz primitiva existe fi(fi(n))raizes primitivaslogo raizes primitivas incongruentes eram raizes que sao potencias de mesma base.gostaria de saber se esta certo???  
		 
Yahoo! doce lar. Faça do Yahoo! sua homepage.

Re: [obm-l] raizes primitivas...

2006-01-10 Por tôpico diego andres
valeu gugu pela sua atençao,vou dar uma olhada no seu artigo. agradecidamente: Diego Andrés. [EMAIL PROTECTED] escreveu:Caro Diego,   Isto não é verdade em geral. Por exemplo, 7 é raiz primitiva módulo 5, masnão é raiz primitiva módulo 25 (sua ordem módulo 25 é 4). O que é verdade éque, se p é um primo ímpar e a é raiz primitiva módulo p^2 então a é raizprimitiva módulo p^k para todo k natural. Veja meu artigo na Eureka 2.   Abraços,GuguCitando diego andres : pra ser mais preciso, a duvida esta nesta parte da soluçao:   se "a" eh raiz primit!
iva
 modulo p,e pelo o teorema de euler vem:   a^(p-1)eh congruente a 1 mod p   a^(p(p-1))eh congruente a 1 mod p^2   a^((p^2)(p-1))eh congruente a 1 mod p^3      a^((p^k)(p-1))eh congruente a 1 mod p^k   logo suponhamos que para um k,a ordem de "a" modulo p^k=j:   assim, p-1 divide j que divide (p^k)(p-1) existem estes j´s que satisfazem:   (p-1),(p(p-1)), ((p^2)(p-1)),...,((p^k)(p-1)) mas como eh que  provo que soh o,((p^k)(p-1)) que satisfaz a congruencia:  a^((p^k)(p-1))eh congruente a 1 mod p^k diego andres  escreveu: Alguem poderia provar pra mim que se "a" eh uma raiz primitiva modulo "p" entao "a" tambem eh uma raiz primitiva de p^w onde "a" eh incongruente a 1 modulo p^2.   Agradecidamente Diego Andrés
 -   Yahoo! doce lar. Faça do Yahoo! sua homepage. -  Yahoo! doce lar. Faça do Yahoo! sua homepage.This message was sent using IMP, the Internet Messaging Program.=Instruções para entrar na lista, sair da lista e usar a lista emhttp://www.mat.puc-rio.br/~nicolau/olimp/obm-l.html=
		 
Yahoo! doce lar. Faça do Yahoo! sua homepage.

Re: [obm-l] raizes primitivas...

2006-01-09 Por tôpico diego andres
pra ser mais preciso, a duvida esta nesta parte da soluçao:  se "a" eh raiz primitiva modulo p,e pelo o teorema de euler vem:  a^(p-1)eh congruente a 1 mod p  a^(p(p-1))eh congruente a 1 mod p^2  a^((p^2)(p-1))eh congruente a 1 mod p^3    a^((p^k)(p-1))eh congruente a 1 mod p^klogo suponhamos que para um k,a ordem de "a" modulo p^k=j:  assim, p-1 divide j que divide (p^k)(p-1) existem estes j´s que satisfazem:  (p-1),(p(p-1)),((p^2)(p-1)),...,((p^k)(p-1)) mas como eh que provo que soh o,((p^k)(p-1))que satisfaz acongruencia: a^((p^k)(p-1))eh congruente a 1 mod p^kdiego andres [EMAIL PROTECTED] escreveu:Alguem poderia provar pra mim que se "a" eh uma raiz primitiva modulo "p" entao"a" tambem eh uma raiz primitiva de p^w onde "a" eh incongruente a 1 modulo p^2.  Agradecidamente Diego Andrés  Yahoo! doce lar. Faça do Yahoo! sua homepage.  
		 
Yahoo! doce lar. Faça do Yahoo! sua homepage.

Re: [obm-l] raizes primitivas...

2006-01-09 Por tôpico diego andres
diego andres [EMAIL PROTECTED] escreveu:deslculpe ai pessoal tow consertando um erro eh:a^((p^(k-1))(p-1) ao inves de a^((p^k)(p-1).  pra ser mais preciso, a duvida esta nesta parte da soluçao:  se "a" eh raiz primitiva modulo p,e pelo o teorema de euler vem:  a^(p-1)eh congruente a 1 mod p  a^(p(p-1))eh congruente a 1 mod p^2  a^((p^2)(p-1))eh congruente a 1 mod p^3    a^((p^(k-1))(p-1))eh congruente a 1 mod p^klogo suponhamos que para um k,a ordem de "a" modulo p^k=j:  assim, p-1 divide j que divide (p^(k-1))(p-1) existem estes j´s que satisfazem: 
 (p-1),(p(p-1)),((p^2)(p-1)),...,((p^k)(p-1)) mas como eh que provo que soh o,((p^k)(p-1))que satisfaz acongruencia: a^((p^(k-1))(p-1))eh congruente a 1 mod p^kdiego andres [EMAIL PROTECTED] escreveu:Alguem poderia provar pra mim que se "a" eh uma raiz primitiva modulo "p" entao"a" tambem eh uma raiz primitiva de p^w onde "a" eh incongruente a 1 modulo p^2.  Agradecidamente Diego Andrés  Yahoo! doce lar. Faça do Yahoo! sua homepage.Yahoo! doce lar. Faça do Yahoo! sua
 homepage.
		 
Yahoo! doce lar. Faça do Yahoo! sua homepage.

[obm-l] teoria dos numeros!!!

2005-12-29 Por tôpico diego andres
alguem me mostra um jeito fácil de descobrir um sistema completo de residuos  modulo 7onde todos os elementos sao primos...  valeu ai pessoal,Diego
		 
Yahoo! doce lar. Faça do Yahoo! sua homepage.

Re: [obm-l] PROBLEMAS LÚDICOS!

2005-12-21 Por tôpico diego andres
seja x o total de livros da biblioteca, y o numero de paradidaticos e w o numero de didaticos.como x=50, e x/5=y, e x/7=w, e como y e w sao inteiros teremos x como multiplo do MMC de 5 e 7 ou seja x =35k mas pela desigualdade inicial x=35 e k=1.Jorge Luis Rodrigues e Silva Luis [EMAIL PROTECTED] escreveu: Ok! Rogério, Matos...muito bem lembrado do artifício extra-simetria...e somente para relaxar...Eduardo possui uma pequena biblioteca, onde guarda seus livros. Após uma ampliação, o número total de livros da biblioteca não passou de 50. Sabe-se que exatamente 20% dos livros da nova biblioteca são didáticos e que exatamente 1/7 do total são paradidáticos. Quantos livros tem a biblioteca (ampliada)?Com quantas pesagens posso identificar o peso de 80 mochilas com p!
rovisões
 e pesos distintos, que são conhecidos por uma lista, usando sòmente uma balança com uma seta indicando a diferença de peso das mochilas dos pratos? (Esse é legal!)Nos jogos estudantis participaram 401 atletas de cinco colégios. Qualquer que seja o grupo de seis atletas que se forme há pelo menos quantos atletas do mesmo colégio, da mesma idade e do mesmo sexo?Se eu tivesse pescado cinco vezes mais do que pesquei, então seriam exatamente tantos acima de 99 como realmente o são abaixo de 99. Quantos peixes pesquei?Quantos são os degraus visíveis de uma escada rolante, se desço 35 passos da escada duas vezes mais rápido do que subo 20 passos? (Apesar de já bastante conhecido, vale a pena ver de novo)À todos, uma boa diversão!_Você sabia que com o seu MSN Messenger você faz ligações de PC-papa- PC, grátis e para qualquer lugar d!
o mundo?
 É só acessar   http://imagine-msn.com/messenger/default2.aspx?locale=pt-br=Instruções para entrar na lista, sair da lista e usar a lista emhttp://www.mat.puc-rio.br/~nicolau/olimp/obm-l.html=
		 
Yahoo! doce lar. Faça do Yahoo! sua homepage.

Re: [obm-l] Data do resultado OBM

2005-12-19 Por tôpico diego andres
já saiu e está no site!!! Marcos Paulo [EMAIL PROTECTED] escreveu: OI,existe alguma previsão de quando sai o resultado da OBM?[]'s MP=Instruções para entrar na lista, sair da lista e usar a lista emhttp://www.mat.puc-rio.br/~nicolau/olimp/obm-l.html=
		 
Yahoo! doce lar. Faça do Yahoo! sua homepage.

Re: [obm-l] inducao

2005-12-17 Por tôpico diego andres
para k=1 é verdadeiro ,pois 1=1/2 para k=n temos 1/k+1 + 1/k+2 + ...+1/2k=1/2(tese) para k=n+1 temos 1/k+2 + 1/k+3 + ...+1/2k+1=1/2(hipótese)pela hipótese,podemos somar 1/k+1 dos dois lados e passar subtraindo 1/2k+1 logo vem: 1/k+1 + 1/k+2 + ...+1/2k=1/2+1/k+1 - 1/2k+1(assim está provado pois obteu-seuma afirmação.) a segunda sai di!
reto
 ,pois uma vez provado a primeira a segunda é consequencia da primeira ja que n é natural.  mas tem um jeito bom de provar isso: todos os termos dessa soma é maior ou igual que 1/2n como se tem n termos podemos: 1/n+1 =1/2n,1/n+2=1/2n, ...,1/2n=1/2n somando as n desigualdades temos: 1/n+1 + 1/n+2 + ...+1/2n = 1/2  Klaus Ferraz [EMAIL PROTECTED] escreveu: Mostre usando inducao que para todo natural n:1/n+1 + 1/n+2 + ...+1/2n = 1/2Mostre que para todo natural n: 1/n + 1/n+1 + 1/n+2 + ...+1/2n = 1/2a primeira dá por inducao só q nao consegui. a segunda não dá.  Yahoo! doce lar. Faça do Yahoo! sua homepage.
		 
Yahoo! doce lar. Faça do Yahoo! sua homepage.

Re: [obm-l] DISCOS

2005-12-17 Por tôpico diego andres
Esse é o princípio da torre de hanoi,e voce pode ver isso explicado bem direitinho em:http://www.obm.org.br/eureka/artigos/recorrencia.pdfDanilo Nascimento [EMAIL PROTECTED] escreveu: ME AJUDEM COM ESSEDispõe-se de 3 pinos e n discos de vidro com um furo no meio, sendo que os discos têm pesos distintos dois a dois. Sabe-se que se um disco de peso maior é colocado sobre um disco de peso menor, então esse se quebra. É proposto o seguinte jogo : "Todos os n discos estão encaixados no primeiro pino, de maneira que olhando de baixo para cima estão em ordem decrescente de peso." Qual é o menor número de movimentos necessários para se passar todos os discos para o terceiro pino, podendo usar o segundo pino (sem quebrar nenhum disco). Yahoo!
 doce lar. Faça do Yahoo! sua homepage.
		 
Yahoo! doce lar. Faça do Yahoo! sua homepage.

Re: [obm-l] UFPE_2006

2005-12-17 Por tôpico diego andres
Essa questão sai direto por regra de três composta.elton francisco ferreira [EMAIL PROTECTED] escreveu:  Para escaparem de uma penitenciária, 10 prisioneirosdecidem cavar um túnel de 450m de comprimento. Em umafuga anterior, 12 prisioneiros cavaram um túnel de270m, trabalhando 6 horas por noite, durante 9 noites.Se os atuais prisioneiros pretendem trabalhar 4 horaspor noite, em quantas noites o túnel ficará pronto?___ Yahoo! doce lar. Faça do Yahoo! sua homepage. http://br.yahoo.com/homepageset.html =Instruções para entrar na l!
ista,
 sair da lista e usar a lista emhttp://www.mat.puc-rio.br/~nicolau/olimp/obm-l.html=__Faça ligações para outros computadores com o novo Yahoo! Messenger http://br.beta.messenger.yahoo.com/ 

Re: [obm-l] numeros primos

2005-12-13 Por tôpico diego andres
observe que : 3^a - 2^b =p , p+2^b=3^alogo p+2^b congruente 1 mod 2o que implica que p eh impar logo o menor p nao representavel eh: 2  Rodrigo Augusto [EMAIL PROTECTED] escreveu: preciso de ajuda com essa questão:Qual o menor número primo P que NAO pode ser representado na forma 3^a - 2^b (em módulo) ? onde a e b são inteiros positivos.por favor, apresentem a resolucao!valeu_MSN Messenger: converse online com seus amigos .  http://messenger.msn.com.br=Instruções para entrar na lista, sair da lista e usar a lista
 emhttp://www.mat.puc-rio.br/~nicolau/olimp/obm-l.html=
		 
Yahoo! doce lar. Faça do Yahoo! sua homepage.

Re: [obm-l] Sequencias e series

2005-12-11 Por tôpico diego andres
Isso sai devido que aparece auma p.g. de razao p ,entao a soma dos n primeiros termos eh : a1*(q^(n)-1)\(q-1) como no caso a1=1,q=p ai vem Sn = (p^(n)-1)/(p-1).Ricardo Serone [EMAIL PROTECTED] escreveu: To precisando de ajuda nos seguintes exercicios:1 - Seja o termo an=p^(n-1), p E R e n E N . Seja, S o somatório dos termos de an de 1 até  + infinito; então demonstre queSn = (p^(n)-1)/(p-1).=Instruções para entrar na lista, sair da lista e usar a lista emhttp://www.mat.puc-rio.br/~nicolau/olimp/obm-l.html=
		 
Yahoo! doce lar. Faça do Yahoo! sua homepage.

Re: [obm-l] Trigonometria

2005-11-23 Por tôpico diego andres
"Igor O.A." [EMAIL PROTECTED] escreveu:caro Igor você não se cofundiu e ao invés de cosseno não é tangente.Achar as soluções de cos(x) . cos(5x) . cos(7x) = [sqrt(3)]/3 (tangente de 30º)  Obrigado.-- I G O RJesus ama você. 
		 
Yahoo! Acesso Grátis: Internet rápida e grátis.Instale o discador agora!

[obm-l] probabilidade(ajuda)!!!

2005-11-17 Por tôpico diego andres
Se a cada 10,1 tem tuberculose temos a probabilidade de 1/10.Desses 8/10 dão positivo no teste logo espaço amostral= (1/10)*(8/10)=8/100.Espaço total vai ser o quanto falta da população ,9/10 ,vezes 3/10 que é a probabilidade de da positivo essa multiplicação mais o espaço amostral logo: (8/100)/(27/100+8/100)=8/35. 
		 
Yahoo! Acesso Grátis: Internet rápida e grátis.Instale o discador agora!

[obm-l] uso inteligente da calculadora!!!

2005-11-17 Por tôpico diego andres
Presuposto:  Para um número ser divisivel por 11 a diferença da soma números de ordem impar com a soma dos de ordem par tem que ser divisivel por 11.Para um número ser divisivel por 9 a soma dos algarismos tem que ser divisivel por 9.  Soma dos de ordem impar=25, soma dos de ordem par=20 para o numero ser divisivel por 11 ou a soma dos de ordem impar =31 ou =20, ou entao a soma dos de ordem par igual =14 ou =25.Mas como o número é 9k+3 vem: soma dos algarismos igual a 9k+3 logo a soma dos de ordem par ficou igual à 14 que é a única possibilidade assim o 8 se transformou no 2(8 é defetuoso), e o numero que apareceu na calculadora foi: 927654321. 
		 
Yahoo! Acesso Grátis: Internet rápida e grátis.Instale o discador agora!

[obm-l] congruencia !!!

2005-11-14 Por tôpico diego andres
Alguém se abilitaria á resolver esta questão para mim: 

Dados "a","c" inteiros positivo e "b" inteiro,prove que existe x inteiro positivo tal que "c" divide a^x+x-b.
		 
Yahoo! Acesso Grátis: Internet rápida e grátis.Instale o discador agora!